Hat der hermitesche Operator H=−d2dx2H=−d2dx2H=-\frac{d^2}{dx^2} imaginäre Eigenwerte?

In der Quantenmechanik spielen hermitesche Operatoren eine sehr wichtige Rolle, da sie reelle Eigenwerte besitzen.

Angesichts D 2 D X 2 , es ist ein hermitescher Operator (eigentlich ist es der einfachste Hamiltonoperator) und seine Eigenfunktion kann ausgedrückt werden durch e ich k X .

Was wäre jedoch, wenn k ist eingebildet? Wenn k ist eingebildet, D 2 D X 2 e ich k X = k 2 e ich k X immer noch gilt, mit seinem Eigenwert k 2 imaginär!

Kann mir jemand sagen, was hier falsch ist?

EDIT: Wie ein Kommentar feststellt, liegt das Problem möglicherweise in der Einschränkung von L 2 . Leider ignorieren die meisten Physik-Lehrbücher einfach die Einschränkung von L 2 . Kann mir jemand einen rigorosen Beweis für diese einfache Welle geben? e ich k X (mit k real) ist in den meisten Situationen irgendwie angemessen?

Siehe auch : physical.stackexchange.com/q/81041/2451 und darin enthaltene Links.
NEIN, e ich k X sind keine Eigenfunktionen, sie liegen nicht darin L 2 . Seit D 2 / D X 2 symmetrisch/hermitesch ist, muss das Spektrum reell sein.
Ja, ich weiß, das Problem kann in der Einschränkung liegen L 2 . Leider ignorieren die meisten Physik-Lehrbücher einfach die Einschränkung von L 2 . Kann mir jemand einen rigorosen Beweis für diese einfache Welle geben? e ich k X reicht irgendwie in den meisten Situationen?
@Martin: Vorsicht mit der Terminologie: Hermitische Operatoren , bei denen die Domänen des Operators und ihrer Adjungierten nicht zusammenfallen, haben echte Eigenwerte , können aber imaginäre Elemente in ihren Spektren haben . Nur für selbstadjungierte Operatoren ist das Spektrum auch reell, und nur für sie haben wir eine Basis aus reellen Eigenvektoren des gesamten Raums.
@ACuriousMind: Richtig. Ich weiß das, hätte aber nicht nachsichtig sein sollen, da diese Frage in erster Linie aus einem zu nachgiebigen Umgang mit der Mathematik entsteht ...

Antworten (2)

Ihre "imaginären Eigenwerte" funktionieren nicht, weil die Eigenfunktionen keine Eigenfunktionen sind. Sie liegen nicht drin L 2 , wie Sie zu wissen scheinen.

Beschäftigen wir uns also mit dem Laplace-Operator selbst: Δ = D 2 D X 2 . Ich möchte die Fourier-Transformation dieses Operators berechnen, da die Fourier-Transformation diagonalisiert Δ , wie wir sehen werden. An der Diagonalform können wir das Spektrum ablesen und daraus schließen, dass das Spektrum besteht aus R + . Natürlich beantworte ich nicht die Frage „warum e ich k X sind echt genug k “, einfach weil es im Hilbert-Raum-Setting eine bedeutungslose Frage ist.

Also machen wir's. Nehmen Sie eine Funktion ψ L 2 ( R ) und rechnen:

F ( Δ ψ ) ( k ) = e ich k X ( D 2 D X 2 ) ψ ( X ) D X = k 2 e ich k X ψ ( X ) D X = k 2 F ( ψ ) ( k )

wo wir zweimal nach Teilen integriert haben (mit that ψ verschwindet notwendigerweise im Unendlichen) und dann differenziert e ich k X . Diese Formel bedeutet das F diagonalisiert den Laplace-Operator, weil wir gerade gesehen haben, dass die Fourier-Transformation des Laplace-Operators F ( Δ ) F ist ein Multiplikationsoperator. Die Idee ist nun, dass wir an einem Multiplikationsoperator das Spektrum ablesen können: Es ist der wesentliche Bereich des Multiplikationsoperators. Nur aus der Definition können wir erkennen, dass dies der Fall sein wird [ 0 , ) in unserem Fall also σ ( Δ ) = [ 0 , ) .

Zu keinem Zeitpunkt haben wir darüber gesprochen e ich k X , also wie man das löst e ich k X mit imaginär k Geschäft? Das müssen Sie auch nicht – keine dieser Funktionen ist drin L 2 , aber: wenn du jetzt reinsteckst ψ ( X ) = e ich k X mit echt k (Weil k in der Fourier-Transformation reell ist!), dann sieht es so aus, als wäre es eine Eigenfunktion von Δ . Mit anderen Worten: Wenn Sie sich einen Raum vorstellen, dessen Basis die Funktionen sind e ich k X , dann ist die Fourier-Transformation des Laplace-Operators nur die unendliche Matrix mit Eigenwerten k 2 auf der Diagonale. Dies ist jedoch nicht das strenge Bild!

Bearbeiten: Eine sehr gute Erklärung mit mehr Mathematik finden Sie hier: https://math.stackexchange.com/questions/766479/what-is-spectrum-for-laplacian-in-mathbbrn Beachten Sie, dass dies keine einfache Mathematik ist, aber Um die Feinheiten dieses Geschäfts zu verstehen, kommt man nicht darum herum.

Das ist eine nette Erklärung und eine gute Ergänzung zu meinem Versuch, wie ich finde.

Erstmal ggf k =: ich κ imaginär ist, ist der Eigenwert („Energie“) κ 2 , dh reell aber negativ:

D 2 D X 2 e ich k X = D 2 D X 2 e κ X = κ 2 e κ X .

Physikalisch ist das eine evaneszente Welle in eine Richtung, wächst aber unbegrenzt in die andere, also wenn Ihr Raum ganz ist X R , ist es aus sehr handfesten physikalischen Gründen keine gültige Wellenfunktion: Es ist nicht normalisierbar. Der zugrunde liegende Grund ist, dass seine Energie kleiner ist als das Minimum des Potentials, in dem es lebt. Jeder einführende QM-Vortrag oder jedes Buch sollte dies behandeln und auch diese Art von Wellenfunktion enthalten (wahrscheinlich im Zusammenhang mit endlichen Potentialbarrieren).

Für die eher formale, mathematische Seite der Dinge sei dieses Dokument empfohlen: http://arxiv.org/abs/quant-ph/9907069 .

Auch die ebenen Wellen sind keine wirklich gültigen Zustände (physikalisch wird ein Teilchen nie einen perfekt definierten Impuls haben), aber sie sind bequem zu handhaben und oft ausreichend (prototypische Situation: ein Teilchenstrahl trifft auf ein Ziel). durch eine ebene Welle beschrieben wird). Wenn eine einzelne ebene Welle nicht gut genug ist (um realistischer zu sein, könnte ein Teilchen als Wellenpaket beschrieben werden), können Sie immer noch eine Zerlegung in ebene Wellen vornehmen, dh eine Fourier-Transformation. Ihr aktueller (wohlerzogener, mathematisch gültiger) Zustand kann als Summe über ebene Wellen beschrieben werden.